AIMO 2015 Q4

A fraction, expressed in its lowest terms a b \dfrac {a}{b} , can also be written as 2 n + 1 n 2 \dfrac {2}{n} + \dfrac {1}{n^2} , where n n is a positive integer.

If a + b = 1024 a+b=1024 , what is the value of a a ?


The answer is 63.

This section requires Javascript.
You are seeing this because something didn't load right. We suggest you, (a) try refreshing the page, (b) enabling javascript if it is disabled on your browser and, finally, (c) loading the non-javascript version of this page . We're sorry about the hassle.

5 solutions

Timothy Westfield
Dec 17, 2015

Since a + b = 1024 a+b=1024 we can say that a b = a 1024 a \frac{a}{b}=\frac{a}{1024-a} leading to a 1024 a = 2 n + 1 n 2 \frac{a}{1024-a}=\frac{2n+1}{n^{2}} .

Cross multiplying gives us a n 2 = 2048 n 2 a n + 1024 a an^{2}=2048n-2an+1024-a .

Bringing all the terms with a a to one side and factoring we get a ( n 2 + 2 n + 1 ) = 1024 ( 2 n + 1 ) a(n^{2}+2n+1)=1024(2n+1) .

Dividing through we get a = 1024 ( 2 n + 1 ) ( n + 1 ) 2 a=\frac{1024(2n+1)}{(n+1)^{2}} .

Because a b \frac{a}{b} is in lowest terms both a a and b b cannot be even. With the sum a + b = 1024 a+b=1024 we can see that both must be odd.

Because a a must be odd 1024 1024 must be completely divided by ( n + 1 ) 2 (n+1)^{2} . Thus 32 n + 1 32|n+1 .

Let k k be the value such that 32 k = n + 1 32k=n+1 , thus n = 32 k 1 n=32k-1 .

Substituting we get a = 1024 ( 64 k 1 ) 1024 k 2 = ( 64 k 1 ) k 2 a=\frac{1024(64k-1)}{1024k^{2}}=\frac{(64k-1)}{k^{2}} .

The only value for which this is an integer is k = 1 k=1 .

Thus n = 31 n=31 and a = 63 a=63 .

Akshat Sharda
Dec 1, 2015

a b = 2 n + 1 n 2 = 2 n + 1 n 2 \frac{a}{b}=\frac{2}{n}+\frac{1}{n^2}=\frac{2n+1}{n^2}

Now we can see that a a and b b are in the form of 2 n + 1 2n+1 and n 2 n^2 respectively.

a + b = 1024 n 2 + 2 n + 1 = 1024 n 2 + 2 n 1023 = 0 \begin{aligned} a+b & =1024 \\ n^2+2n+1 & =1024 \\ n^2+2n-1023 & =0 \end{aligned}

Now we can solve the above quadratic equation and we'll get n = 33 , 31 n=-33,31 . But 33 -33 will be rejected as n n is a positive integer, so, n = 31 n=31 .

Placing n = 31 n=31 in 2 n + 1 n 2 \frac{2n+1}{n^2} will give us the value of a a as 2 n + 1 = 63 2n+1=\boxed{63} .

Moderator note:

There is a slight leap in going from a b = 2 n + 1 n 2 \frac{a}{b} = \frac{ 2n+1} { n^2} and claiming that a = 2 n + 1 a = 2n+1 and b = n 2 b = n^2 .

For example, since we have 3 4 = 6 8 \frac{3}{4} = \frac{6}{8} , can we conclude that 3 = 4 3 = 4 and 6 = 8 6 = 8 ? If not, what is so special about this scenario?

There is a slight leap in going from a b = 2 n + 1 n 2 \frac{a}{b} = \frac{ 2n+1} { n^2} and claiming that a = 2 n + 1 a = 2n+1 and b = n 2 b = n^2 .

For example, since we have 3 4 = 6 8 \frac{3}{4} = \frac{6}{8} , can we conclude that 3 = 4 3 = 4 and 6 = 8 6 = 8 ? If not, what is so special about this scenario?

Calvin Lin Staff - 5 years, 6 months ago

Log in to reply

The reason is that n^2&2n+1 are coprime

himanshu singh - 5 years, 5 months ago

Log in to reply

Indeed! And of course, we should show why:

( \gcd(2n+1, n^2 ) \leq \gcd( 2n+1, 4n^2) \leq \gcd ( (2n+1)(2n-1) , 4n^2) = \gcd (4n^2 -1 , 4n^2) = \gcd(1, 4n^2) ]

Calvin Lin Staff - 5 years, 5 months ago
Reese Wilkinson
Dec 4, 2015

Rewrite the n terms into one fraction; a b = 2 n + 1 n 2 \frac {a}{b}=\frac {2n +1}{n^{2}} So we know that a is always odd, so for a + b = 1024 = 2 10 a+b=1024= 2 ^{10} to hold true, b is odd. Thus in our case,

n = 2 k + 1 : k N 0 n=2k +1 : k\in\mathbb {N_{0}}

So we have

a = 4 k + 3 a=4k + 3 b = 4 k 2 + 4 k + 1 b=4k^{2} +4k + 1

Thus,

a + b = 4 k + 3 + 4 k 2 + 4 k + 1 = 2 10 a+b=4k +3+4k^{2} +4k + 1=2^{10} k 2 + 2 k + 1 = 2 8 k^{2} +2k + 1 =2^{8} ( k + 1 ) 2 = 2 8 (k+1)^{2}=2^{8} So k = 15 k=15 n = 31 n=31 a = 63 a=63

There is a slight leap in going from a b = 2 n + 1 n 2 \frac{a}{b} = \frac{ 2n+1} { n^2} and claiming that a = 2 n + 1 a = 2n+1 and b = n 2 b = n^2 .

For example, since we have 3 4 = 6 8 \frac{3}{4} = \frac{6}{8} , can we conclude that 3 = 4 3 = 4 and 6 = 8 6 = 8 ? If not, what is so special about this scenario?

Calvin Lin Staff - 5 years, 6 months ago

It is given that:

2 n + 1 n 2 = a b and that a + b = 1024 2 n + 1 n 2 = 1024 b b 2 n + 1 n 2 = 1024 b 1 2 n + 1 n 2 + 1 = 1024 b 2 n + 1 + n 2 n 2 = 1024 b ( n + 1 n ) 2 = ( 32 b ) 2 \begin{aligned} \frac{2}{n} + \frac{1}{n^2} & = \frac{a}{b} \quad \quad \small \color{#3D99F6}{\text{and that } a+b = 1024} \\ \frac{2n + 1}{n^2} & = \frac{1024-b}{b} \\ \frac{2n + 1}{n^2} & = \frac{1024}{b} - 1 \\ \frac{2n + 1}{n^2} + 1 & = \frac{1024}{b} \\ \frac{2n + 1 + n^2}{n^2} & = \frac{1024}{b} \\ \left(\frac{n + 1}{n}\right)^2 & = \left(\frac{32}{\sqrt{b}}\right)^2 \end{aligned}

n = 31 and b = 3 1 2 = 961 a = 1024 961 = 63 \Rightarrow n = 31 \text{ and } b = 31^2 = 961 \quad \Rightarrow a = 1024 - 961 = \boxed{63}

Moderator note:

There is a huge leap in going from ( n + 1 n ) 2 = ( 32 b ) 2 \left( \frac{n+1}{n} \right)^2 = \left( \frac{32}{ \sqrt{b} } \right)^2 to n = 31 , b = 3 1 2 n = 31, b = 31^2 . In particular, that is not the only solution. We could have b = 1 6 2 , n = 1 b = 16^2 , n = 1 .

There is a huge leap in going from ( n + 1 n ) 2 = ( 32 b ) 2 \left( \frac{n+1}{n} \right)^2 = \left( \frac{32}{ \sqrt{b} } \right)^2 to n = 31 , b = 3 1 2 n = 31, b = 31^2 . In particular, that is not the only solution. We could have b = 1 6 2 , n = 1 b = 16^2 , n = 1 .

Calvin Lin Staff - 5 years, 6 months ago
Chenjia Lin
Dec 3, 2015

First, add together 2 n \frac{2}{n} and 1 n 2 \frac{1}{n^{2}} to get a single fraction. Given that a b \frac{a}{b} is the reduced form of 2 n \frac{2}{n} + 1 n 2 \frac{1}{n^2} , 2 n \frac{2}{n} + 1 n 2 \frac{1}{n^2} = a b \frac{a}{b} .

2 n \frac{2}{n} + 1 n 2 \frac{1}{n^2} = a b \frac{a}{b}

2 n + 1 n 2 \frac{2n+1}{n^{2}} = a b \frac{a}{b}

Substituting the numerators and denominators into a and b in a + b = 1024 a+b=1024 , we get n 2 + 2 n + 1 = 1024 n^{2} + 2n +1 =1024 . This is a quadratic that we can solve, and factoring this equation, we now have ( n 31 ) ( n + 33 ) = 1024 (n-31)(n+33)=1024 .

Finally, we solve for the n values, and we get n = 31 n=31 and n = 33 n=-33 . But which n value is the one we want? Well, if we plug the two values into 2 n + 1 n 2 \frac{2n+1}{n^{2}} = a b \frac{a}{b} , only n = 31 n=31 will yield a positive a. value (stated in the problem). Thus, a = 2 ( 31 ) + 1 = 63 a=2(31)+1= \boxed{63} .

Moderator note:

There is a slight leap in going from a b = 2 n + 1 n 2 \frac{a}{b} = \frac{ 2n+1} { n^2} and claiming that a = 2 n + 1 a = 2n+1 and b = n 2 b = n^2 .

For example, since we have 3 4 = 6 8 \frac{3}{4} = \frac{6}{8} , can we conclude that 3 = 4 3 = 4 and 6 = 8 6 = 8 ? If not, what is so special about this scenario?

There is a slight leap in going from a b = 2 n + 1 n 2 \frac{a}{b} = \frac{ 2n+1} { n^2} and claiming that a = 2 n + 1 a = 2n+1 and b = n 2 b = n^2 .

For example, since we have 3 4 = 6 8 \frac{3}{4} = \frac{6}{8} , can we conclude that 3 = 4 3 = 4 and 6 = 8 6 = 8 ? If not, what is so special about this scenario?

Calvin Lin Staff - 5 years, 6 months ago

Log in to reply

Is this valid?

Because "a" is the reduced form of 2 n + 1 2n+1 and "b" is the reduced form of n 2 n^{2} , it can be assumed that a = 2 n + 1 a=2n+1 and b = n 2 b=n^{2} .

Chenjia Lin - 5 years, 6 months ago

Log in to reply

Not quite. You are not at the heart of the matter as yet.

The key is to explain that 2 n + 1 n 2 \frac{2n+1} { n^2} is in reduced form (and both are positive). That is the implicit assumption (IE What is so special about this scenario?) in order for us to conclude that the numerators and denominators are identical.

Calvin Lin Staff - 5 years, 6 months ago

Sir, we have g c d ( 2 n + 1 , n 2 ) = 1 gcd(2n+1,n^2)=1 . So, we can assume that a = 2 n + 1 , b = n 2 a=2n+1, b=n^2

Mohammed Imran - 1 year, 2 months ago

Log in to reply

Yup!

Calvin Lin Staff - 1 year, 1 month ago

Thank you sir

Mohammed Imran - 1 year, 1 month ago

0 pending reports

×

Problem Loading...

Note Loading...

Set Loading...